¿Tiene |⟨p|ψ⟩|2|⟨p|ψ⟩|2\lvert\langle p\lvert\psi\rangle\rvert^2 algún significado?

solía pensar | pag | ψ | 2 tenía el significado de alguna probabilidad de que el impulso de la partícula fuera pag (dentro de algún intervalo de tolerancia Δ pag ). Ahora solo estoy confundido.

usaré el impulso pag , número de onda k y velocidad v indistintamente aquí (es decir, usaré la convención metro = 1 , = 1 ). Comienzo con la distribución del impulso en el momento t = 0 . Además, limitémonos a las funciones de onda que inicialmente son reales y también describamos un estado estacionario en alguna configuración de pozo potencial.

Quiero comparar la evolución del estado anterior, con la evolución del estado de una partícula libre. Tan pronto como el reloj avanza un instante más allá t = 0 , estos dos sistemas diferentes evolucionan de manera completamente diferente.

Creo en una teoría que permite que la distribución de probabilidad de las velocidades dependa de ψ t en el momento t = 0 tendría más sentido que una teoría por la cual la distribución anterior depende solo de la forma inicial de la función de onda ψ 0 .

A) Para ejemplificar la evolución de una partícula libre, considere la propagación de un paquete de ondas con el estado inicial ψ 0 ( X ) = Exp ( | X | ) ,

(1) ψ ( X , t ) d k Exp ( i k X i ω k t ) k 2 + 1 ,
dónde ω k = k 2 / 2 .

B) Sin embargo, si el mismo estado inicial describiría la partícula en el pozo, la partícula se encuentra en un estado estacionario ligado, y | ψ ( X , t ) | 2 simplemente permanece constante en el tiempo. Nada parecido a la evolución. (1) .

Discusión : mi punto era elegir un estado inicial (real, sin corriente) y mostrar que hay una gran cantidad de corriente en muy poco tiempo , es decir, me parece que la diferencia entre | ψ A ( X , t ) | 2 y | ψ B ( X , t ) | 2 ocurre casi instantáneamente .

En el estado (1) no tenemos corriente de probabilidad, j ( X , t ) = 0 . (La corriente viene dada por la densidad de probabilidad multiplicada por la velocidad de Bohm j ( X , t ) = | ψ | 2 v promedio , dónde v promedio = metro X argumento ( ψ ) es una cantidad central en las interpretaciones bohmianas de QM.) Pero la corriente cero no significa que no haya ningún movimiento, simplemente la velocidad de las partículas que se mueven hacia el brillante contrarresta la velocidad de las partículas que van hacia la izquierda. Y me cuesta creer que la distribución de velocidades en los casos A y B sea exactamente igual.

(Hay problemas de pozos similares, uno de ellos es la distribución ψ 0 = Exp ( X 2 ) y en comparación con una partícula en un oscilador armónico de estado fundamental. Este sería mejor para usar si queremos hablar sobre la distribución de velocidad y la energía cinética. Y, dado que hay estados de energía de estado límite más altos, sobre el límite clásico).

C) Un pensamiento adicional: en el límite clásico (gran número de nodos dentro de algún espacio Δ X ), se debería poder hablar de una distribución de velocidades cerca de una posición X . debería ser algo como 0.5 d ( v v C ) + 0.5 d ( v + v C ) dónde v C = 2 T / metro . ¿Cómo se compara esta distribución con cualquier significado que podamos obtener de la distribución sin posición? | pag | ψ | 2 ? Pensando clásicamente, creo que un factor d t veces la diferencia de aceleración entre los dos sistemas debería explicar la diferencia de evolución.

¿Habría alguna forma de obtener una distribución de velocidad dependiente de la posición, o incluso una cantidad similar a esta, que coincidiera con el límite clásico?

Respuestas (4)

solía pensar | pag | ψ | 2 tenía el significado de alguna probabilidad de que el impulso de la partícula fuera pag (dentro de algún intervalo de tolerancia Δ pag ). Ahora solo estoy confundido.

ρ ( pag ) = | pag | ψ | 2 es solo otra notación para | ψ ~ ( pag ) | 2 dónde

ψ ~ ( pag ) = 1 2 π ψ ( q ) mi i pag q d q .

Se supone (y matemáticamente funciona bien) que así como q 1 q 2 | ψ ( q ) | 2 d q da probabilidad de que q ( q 1 , q 2 ) ,

pag 1 pag 2 | ψ ~ ( pag ) | 2 d pag

da probabilidad de que pag ( pag 1 , pag 2 ) -

Sin embargo, me cuesta creer que la distribución de velocidades de estos dos casos sea exactamente igual.

Tómelo como una receta para calcular la distribución de velocidades de la mejor manera que podamos. No refleja necesariamente las velocidades reales.

En el límite clásico (gran número de nodos dentro de algún espacio D X ), se debería poder hablar de una distribución de velocidades cerca de una posición X . debería ser algo como 0.5 d ( v v C ) + 0.5 d ( v + v C ) dónde v C = 2 T / metro . ¿Cómo se compara esta distribución con cualquier significado que podamos obtener de la distribución sin posición? | pag | ψ | 2 ?

Si usted tiene q -distribución de probabilidad dependiente para pag , siempre puede calcular la distribución de probabilidad marginal para pag (sin tener en cuenta el valor de q ) como

m ( pag ) = ρ q ( pag ) d q
y compararlo con ρ ( pag ) . Si m ( pag ) y ρ ( pag ) no está de acuerdo, entonces puede pensar por qué es eso y cuál prefiere como una mejor descripción.

No le respondiste por qué la función de onda evoluciona en el caso A de manera diferente que en el caso B. Ambos son casos cuánticos. Él cree que una y la misma función de onda puede ser una función de onda inicial (de lo cual no estoy seguro, pero déjalo así). Y pregunta por qué la distribución de velocidades evolucionará de manera diferente.
@Sofia, no vi ninguna de esas preguntas en la primera publicación.
Está bien, puse un comentario que completa el problema.
Creo que esto ayuda, lo que estoy buscando es un ρ q ( pag ) que tendría que estar de acuerdo con ρ ( pag ) = | pag | ψ | cuando se integra sobre q como arriba.

Lo que nos impide hablar de la probabilidad de encontrar una partícula con cantidad de movimiento pag en la posición X en mecánica cuántica? Por supuesto, es la no conmutatividad de X ^ y pag ^ , o en términos más físicos, el principio de incertidumbre de Heisenberg

Δ X Δ pag 2 .
Esta es la razón por la cual la función de Wigner mencionada por Void no es una distribución de probabilidad no negativa adecuada.

Pero tratemos de minimizar el efecto del principio de Heisenberg. Recordar que

I = ϕ ( X ) ψ ( X ) d X
es una medida de la superposición de los estados ϕ y ψ . En particular, la probabilidad de encontrar el estado ϕ es PAG = | I | 2 . Ahora, considere el siguiente estado
ϕ ( X ; pag , X ) = C 1 Exp [ C 2 ( X X ) 2 + i pag X ]
dónde C 1 y C 2 son algunas constantes de normalización sin importancia. No es demasiado difícil demostrar que los valores esperados de X ^ y pag ^ en este estado se encuentran X y pag . Además, para este estado Δ X Δ pag = / 2 , lo que significa que la incertidumbre es mínima . Defina así la función de Husimi
q ( X , pag ) = | ϕ ( X ; X , pag ) ψ ( X ) d X | 2
que luego da la probabilidad de encontrar un estado de incertidumbre mínima con cantidad de movimiento pag en X .

La función de Husimi se puede relacionar con la función de Wigner y viceversa, y se puede usar para calcular cualquier cosa para la que se pueda usar la función de Wigner.

Su explicación es mil veces más fácil de entender que lo que pude encontrar en wiki (en realidad, nunca he tenido distros muy raros, wiki fue mi única fuente, ¡y no había oído hablar de Husimi hasta hoy!). Parece un producto interno con un gaussiano en movimiento/modulado, y eso debería ponderar intuitivamente el impulso más a lo que está cerca de x'. (No estoy seguro si falta un * en el ϕ .) Este Q(x',p) realmente parece ser lo que estoy buscando; mil millones de gracias
+1, no pensé en esto. Cabe señalar que este q ( X , pag ) no es único. Tenemos C 2 = 1 / ( 4 ( Δ X ) 2 ) = ( Δ pag ) 2 ( = 1 ); eligiendo diferentes Δ pag usará una base diferente y obtendrá una q ( X , pag ) . Cuando tomas Δ pag 0 , usted tendrá q ( X , pag ) ρ ( pag ) y cuando Δ pag conseguirás q ( X , pag ) ρ ( X ) . La función Husimi es útil si tiene una base bien definida sobre la cual proyectar su función de onda y puede usarse como una herramienta para obtener el límite casi clásico, pero debe tener cuidado con su ambigüedad.

Interpretando la corriente de probabilidad j como una corriente real que determina una especie de velocidad bohmiana de una corriente de fluido (conjunto estadístico) no se puede mezclar con la interpretación habitual de la mecánica cuántica, lo más importante es que no se puede mezclar con el principio de superposición.

Considere el siguiente ejemplo: tomamos una función de onda mi i k X , esta función de onda representaría un conjunto de transmisión de velocidad k en el cuadro de Bohm ( = metro = 1 ). Superpongámoslo ahora con un conjunto de velocidad k , es decir mi i k X , de modo que obtenemos partículas en contracorriente. Eso significa que, desde el punto de vista de Bohm, obtendremos dos conjuntos de transmisión contraria que dan una velocidad cero y una distribución de probabilidad espacial plana, ¿verdad? No exactamente. Nuestra función de onda resultante es

ψ ± k porque ( k X )
con la distribución de probabilidad 1 + C o s ( 2 k X ) . De repente, las trayectorias bohmianas no fluyen pacíficamente, sino que se congelan entre los nodos de porque ( k X ) . ¡Esta es una imagen muy diferente de lo que daría una interpretación ingenua al estilo de Bohm! No hay forma de que las partículas fluyan a través de los nodos de probabilidad cero de partículas, por lo que solo podemos decir que las partículas están oscilando de alguna manera entre los nodos de porque ( k X ) .

De cualquier manera, puede ver que combinar el concepto de un mi i k X El conjunto de transmisión con superposición no da nada satisfactorio. La superposición y una interpretación "beable" de la función de onda simplemente no funcionan juntas. Los "Beables" no son lineales y todas las preguntas "lineales" no proporcionarán una buena respuesta.

Tomando pag | ψ con una mentalidad similar a la de Bohm es como preguntar "¿Cuánto de un uniforme pag -arroyo ( mi i pag X ) está linealmente presente en este conjunto ψ ?" y obtienes una respuesta matemática que no tiene un buen significado en el sentido de la física no lineal que crees que es verdadera. Solo obtienes una transformada de Fourier útil de esta función peculiar ψ de tu teoría.


Hay una función de "cuasi probabilidad" en el espacio de fase que me parece bastante ordenada, la distribución de casi probabilidad de Wigner PAG ( pag , X ) . Tiene todas las buenas propiedades que esperaría de una distribución de este tipo, como PAG ( pag , X ) d pag = ρ ( pag ) , PAG ( pag , X ) d X = ρ ( X ) . Es realmente genial, pero... no es definitivamente positivo. Sin embargo, a veces se usa en el límite clásico.


La verdad es que el estado cuántico no puede interpretarse en términos de objetos clásicos, pero debe entenderse en sus propios términos.

La verdadera contraparte clásica del estado cuántico es la posición nítida y el momento de la partícula, porque el estado cuántico es el componente básico de su teoría, lo más nítido que puede obtener, la caracterización completa del sistema y su evolución. Y la interpretación y convergencia al límite clásico debe hacerse solo verificando la correspondencia entre X X ^ , ( X ^ ) 2 ( X ^ ) 2 0 etc.

Por otro lado, la contraparte cuántica de una distribución clásica es el operador de densidad . En la mecánica clásica tienes una distribución sobre los objetos nítidos de tu teoría, la posición y los momentos. En la teoría cuántica también, la única diferencia es que tienes una distribución sobre objetos cuánticos nítidos, los estados. A medida que los estados convergen en posiciones y momentos definidos, su operador de densidad converge en una densidad de probabilidad clásica.


¿Estoy maldito o tardo demasiado en completar una respuesta para que otra sea aceptada mientras tanto? De todos modos, disfruta de un punto de vista alternativo.

Creo que en cualquier interpretación siempre tendrás estos efectos de interferencia; Los efectos de fase e interferencia parecen ser el principal impulsor de los fenómenos cuánticos. Para ondas estacionarias sin corriente, los nodos de wavefn le brindan mucha información sobre la fase; si piensas en ψ como superposición de dos corrientes de igual magnitud y dirección opuesta, ψ hacia la derecha y ψ Hacia la izquierda = ψ hacia la derecha , te dice que la fase de estos componentes está en ± π en los nodos (con la rara excepción del caso donde los componentes tienen magnitud 0).

La evolución temporal se puede analizar aplicando el operador de evolución temporal, es decir

tu ^ = mi i H ^ t

Dado que el resultado de H ^ | ψ es diferente en los dos casos que ha establecido (el hamiltoniano que describe los problemas es diferente), los estados evolucionan de manera diferente. El valor esperado para el impulso reflejará esta situación, es decir

pag ^ ( t ) = ψ ( t ) | pag ^ | ψ ( t )

tendrá diferentes dependencias en los dos casos, por lo que la distribución de los momentos cambiará a medida que el valor esperado pag ^ ( t ) cambios.

OP sabe cómo usar el operador de evolución como es obvio a partir de la ecuación. (1). Además, pag es igual en este caso, que es un poco el punto de la pregunta.
@Void Tengo que retirar lo que dije sobre la fórmula (1). Lo siento, es correcto.